Difference between revisions of "2011 AMC 12A Problems/Problem 2"

(Solution)
m (minor edit)
Line 1: Line 1:
 
== Problem ==
 
== Problem ==
 
There are <math>5</math> coins placed flat on a table according to the figure. What is the order of the coins from top to bottom?
 
There are <math>5</math> coins placed flat on a table according to the figure. What is the order of the coins from top to bottom?
 +
 
<asy>
 
<asy>
 
size(100); defaultpen(linewidth(.8pt)+fontsize(8pt));
 
size(100); defaultpen(linewidth(.8pt)+fontsize(8pt));
Line 8: Line 9:
 
draw(arc((-.588,-.809), 1.2, 75, 300));
 
draw(arc((-.588,-.809), 1.2, 75, 300));
 
draw(arc((-.951,.309), 1.2, 96, 228));
 
draw(arc((-.951,.309), 1.2, 96, 228));
label("$A$",(0,1),NW); label("$B$",(-1.1,.309),NW); label("$C$",(.951,.309),E); label("$D$",(-.588,-.809),W); label("$E$",(.588,-.809),S);</asy>
+
label("$A$",(0,1),NW); label("$B$",(-1.1,.309),NW); label("$C$",(.951,.309),E); label("$D$",(-.588,-.809),W); label("$E$",(.588,-.809),S);
<math>
+
</asy>
\textbf{(A)}\ (C, A, E, D, B) \qquad
+
 
\textbf{(B)}\ (C, A, D, E, B) \qquad
+
<math>\textbf{(A)}\ (C, A, E, D, B) \qquad \textbf{(B)}\ (C, A, D, E, B) \qquad \textbf{(C)}\ (C, D, E, A, B) \qquad \textbf{(D)}\ (C, E, A, D, B) \qquad \textbf{(E)}\ (C, E, D, A, B) </math>
\textbf{(C)}\ (C, D, E, A, B) \qquad
 
\textbf{(D)}\ (C, E, A, D, B) \qquad \\
 
\textbf{(E)}\ (C, E, D, A, B) </math>
 
  
 
== Solution ==
 
== Solution ==

Revision as of 01:06, 19 October 2020

Problem

There are $5$ coins placed flat on a table according to the figure. What is the order of the coins from top to bottom?

[asy] size(100); defaultpen(linewidth(.8pt)+fontsize(8pt)); draw(arc((0,1), 1.2, 25, 214)); draw(arc((.951,.309), 1.2, 0, 360)); draw(arc((.588,-.809), 1.2, 132, 370)); draw(arc((-.588,-.809), 1.2, 75, 300)); draw(arc((-.951,.309), 1.2, 96, 228)); label("$A$",(0,1),NW); label("$B$",(-1.1,.309),NW); label("$C$",(.951,.309),E); label("$D$",(-.588,-.809),W); label("$E$",(.588,-.809),S); [/asy]

$\textbf{(A)}\ (C, A, E, D, B) \qquad \textbf{(B)}\ (C, A, D, E, B) \qquad \textbf{(C)}\ (C, D, E, A, B) \qquad \textbf{(D)}\ (C, E, A, D, B) \qquad \textbf{(E)}\ (C, E, D, A, B)$

Solution

By careful inspection and common sense, the answer is $\textbf{(E)}$.

See also

2011 AMC 12A (ProblemsAnswer KeyResources)
Preceded by
Problem 1
Followed by
Problem 3
1 2 3 4 5 6 7 8 9 10 11 12 13 14 15 16 17 18 19 20 21 22 23 24 25
All AMC 12 Problems and Solutions

The problems on this page are copyrighted by the Mathematical Association of America's American Mathematics Competitions. AMC logo.png